LSAT and Law School Admissions Forum

Get expert LSAT preparation and law school admissions advice from PowerScore Test Preparation.

 Adam Tyson
PowerScore Staff
  • PowerScore Staff
  • Posts: 5153
  • Joined: Apr 14, 2011
|
#29476
I think you are reading into the stimulus claims that are not there. The author never says that an effective campaign will lead to higher sales, but only that the particular campaign under discussion would be ineffective and would not lead to higher sales. Don't make a mistaken negation yourself in attempting to describe the author's flaw! Your analysis of both D and E appears to be based on your own mistaken negation.

Ill conceived means bad, ineffective, not well done, so it is indeed referring to LRG getting bad advice.

There is some reasonable debate here about who this competitor is that we are referring to in answer A - a competitor of LRG, or the competing consultant that the author referred to in the stimulus. Let's give the benefit of the doubt here and assume that it is the latter, and that Answer A is referring to the ad campaign that LRG went with against the advice of our author.

In that case, Answer A might indeed describe a problem with the argument, similar to what we often see in Resolve the Paradox questions about a surprisingly bad result (the resolution being that things would have been even worse if not for the X factor in the stimulus, such as the fantastic detective or doctor that always gets the worst cases having the worst success record but still being considered the best in the business). Our author may have assumed, without justification, that the ad campaign did not help boost sales. But then again, he didn't argue that the ad campaign would not help sales relative to doing nothing - he argued that the campaign would not be effective overall, and he went on to tell us that sales were especially poor after the campaign ran. Sounds to me like an ineffective ad campaign, regardless of what might have been worse. So if A is a flaw, it's not a big one, because it isn't really central to his argument.

Now remember, we are in the business on the LSAT of picking not good answers, or acceptable answers, but the BEST answer (of the five presented). Is A a flaw? Yes, maybe, somewhat. Is it the biggest problem? No way, not by far. B describes the big gaping hole in the argument, the failure to consider alternate causes for the poor sales.

For those of you who second-guessed your alternate cause prephrase, don't do that. B is a perfect match and ever so much more powerful than whatever is going on with A. The authors pulled a fast one on you there, but if you go in with a strong mind and a solid prephrase you should not fall prey to their tricks. Sure, keep A as a contender, but when you get to B you should absolutely love it and throw A onto the junk pile pretty confidently.
 Khodi7531
  • Posts: 116
  • Joined: Mar 14, 2018
|
#46330
My question about this is between A and B. I loved A...and still do.


I thought A was getting at a mistaken assumption of the author....that the ignoring of his predictions and advice of the competing consultant is what CAUSED the loss of money and ultimately for him to conclude that it was ill conceived. I thought A got at that perfectly. Discussing causality, that he's assuming the sales wouldn't be as low as they are if they didn't go with the advertising campaign they chose.


Sounds pretty damn good to me.


I hesitated on B because I even though it was bringing in another factor to justify this outcome, I was thinking.. "economic factors?" when was the stimulus alluding to that in anyway. I get to weaken it you can just say it, but when an answer like A get's at the issue of it more, I mean idk how you can circle B.


Admin please respond cause this question is driving me nuts


___________________________________________
Ok wait...is this not correct because even with this being the case, sales being lower or not, the advertising campaign can still be ill conceived?


B is classic in that it shows alternative cause. But this is a flaw question? If it's flaw, isn't an answer supposed to point out what the issue is....instead of weakening it like B does?


HELP AGAIN!!!
 deck1134
  • Posts: 160
  • Joined: Jun 11, 2018
|
#49728
Hi Adam et al,

I thought that this was maybe the hardest question on the test because it seems really poorly written.

Adam said, "
There is some reasonable debate here about who this competitor is that we are referring to in answer A - a competitor of LRG, or the competing consultant that the author referred to in the stimulus. Let's give the benefit of the doubt here and assume that it is the latter, and that Answer A is referring to the ad campaign that LRG went with against the advice of our author
I knew that we were looking for a flaw answer, and thought that there could be an alternate cause. But answer (A) is really a flaw that we would usually see in a Resolve the Paradox argument, that things would be much worse if not for some factor/element/situation. Isn't that a flaw? Say that there was lead found in children's toy paint that LRG uses. Maybe the "competitor" pulled off a miracle by convincing anyone to buy toys! That certainly is a flaw in the argument. While Adam says that it isn't about "relative to nothing," who is to say that the campaign is ineffective if instead of selling 5 toys, they sold 1000. Sure, that may be less than a nationwide average of 1,000,000, but it is still impressive.

I'll concede that (B) is also a flaw It's a strong flaw. But I took this test, saw (A) and (B) in light of the above, and thought that they were both good, solid clear flaws. How the heck do you mediate between them?

Am I right in thinking that this is a bad question? This was the only LR I missed on this test. Not sure how to get it right.
 Adam Tyson
PowerScore Staff
  • PowerScore Staff
  • Posts: 5153
  • Joined: Apr 14, 2011
|
#49756
Khodi - the reason answer A is not the flaw is because the author doesn't argue that the campaign they went with was the worst possible campaign, but only that it was ill conceived. Not that it didn't do any good, an absolute claim, but that it wasn't very good, a relative claim. If he had argued that it was completely without merit and did nothing to help, then answer A would be much better.

Answer B does describe a Flaw rather than Weaken the argument, because it doesn't say that there WERE economic reasons for the poor sales, but only that the author FAILED TO CONSIDER that possibility. Flaw answers merely describe what is wrong, while Weaken answers attack and take advantage of the flaw.

I don't think it's poorly written, Deck, but that's just my opinion. In fact, the fact that it proved so challenging, but still clearly has one right answer and four wrong ones, suggests that it was very well written to achieve their goal of challenging students to see who rises to the occasion!
 s_takesonLSAT
  • Posts: 3
  • Joined: Dec 30, 2019
|
#76402
I wanted to check my reasoning on this question:

During the exam I couldn't decide between A and B and ended up choosing B. Below is my logic:

A is saying that the advertisement helped because without it sales would have been worse. But the marketing consultant is arguing that the campaign wasn’t successful in raising sales and replying with "it could have been worse" really isn’t the best response unless you have more to add.
Even if the sales would have been lower without the campaign why? something had to happen for you to lose people because you didn't run a campaign. If the stimulus said sales did not increase, then yeah sure maybe, but that's not what it said, it says sales dropped. An advertisement doesn't normally lose people unless it maybe it did something offensive, the product your selling is no longer useful, or something happened to the overall economy that has also affected your business. Therefore with answer choice A, we have to make a ton of assumptions. For example, they could say "hey our sales would have been lower without this campaign because of Corona. However, with this campaign, we were able to reach more people. Our average consumers normally were low income, now because of Corona, they do not have a large amount of disposable income to spend on more clothes so this advertisement helped us reach people with higher levels of disposable income."

So no matter what, we have to add another assumption(assumption in my example is something happened that affected everyone). B could support A, but A standing alone is difficult.

I didn't catch the difference in referring to the advertisement as the "competitor's advertisement campaign" being different than the competing consultant. Does my reasoning still work?
 Frank Peter
PowerScore Staff
  • PowerScore Staff
  • Posts: 99
  • Joined: May 14, 2020
|
#76442
Hi S,

It sounds like you're on the right track. One strategy I like to use when stuck between two answer choices like (A) and (B) is to consider how the author might respond in an actual argument. And really, what (A) is saying is immaterial to the author's reasoning. (A) is saying that the campaign wasn't a total failure - that if we compared what the results might have been had there been no campaign, the advertising campaign had at least some effect in generating some sales. But to this, the author would probably respond that raising this point doesn't take away from the fact that it was still an ill conceived campaign, and sales would have been much stronger with a well-conceived campaign.

(B) ends up being the stronger response because the author hasn't considered the possibility that there could be an alternate cause for the low sales figures. Raising this point makes it more doubtful that we can say for certain that it was in fact the ill-conceived campaign that caused the low sales figures.
User avatar
 mab9178
  • Posts: 96
  • Joined: May 02, 2022
|
#95360
Hi,

Answer-choice A states: "it takes for granted that LRG's sales would not have been lower still in the absence of the competitor's advertising campaign."


If answer-choice A had taken out the word "competitor's," and stated as the result of this alteration, "it takes for granted that LRG's sales would not have been lower still in the absence of the advertising campaign," it would've been correct, I think, because it would've attached the supposed cause, "the ad campaign."

In other words, in the absence of the cause, i.e. the ad campaign, sales would NOT have suffered; but this is an unproven assumption taken for granted by the MC.

Am I correct?
User avatar
 katehos
PowerScore Staff
  • PowerScore Staff
  • Posts: 184
  • Joined: Mar 31, 2022
|
#95385
Hi mab9178!

The way I see it, taking out the word 'competitor's' doesn't change the answer. One would reasonably understand that the 'ad campaign' in question would still be the ill-conceived one of the competitor, and, even if they didn't, taking out the word competitor doesn't mean that sales would NOT have suffered without the campaign. Rather, sales, like the answer choice says, might have have suffered MORE ('been lower still') without the campaign.

I hope that makes sense!! :)
-Kate
User avatar
 mab9178
  • Posts: 96
  • Joined: May 02, 2022
|
#95396
Hi Kate,

Thank you for your input.

Because this is an identify the flaw or the error in reasoning, the word "competitor's" knocks out A rather quickly, because the stimulus implies that the "competitor" weighed in favor of the ad that was opposed by the consultant (author of the argument.)

But since based on the stimulus, we do not know whether the competing consultant designed/produced an ad (for all we know the company made an ad and both consultants weighed in on it in opposing opinions), we can eliminate A quickly.

But my question concerns answer-choice A being stated without the word "competitor's." To this you say, "taking out the word competitor doesn't mean that sales would NOT have suffered without the campaign." Well, yes, that IS EXACTLY my point, for without the word "competitor's" A would have stated: "It takes for granted that LRG's sales would not have been lower still in the absence of the advertising campaign."

In other words, without the word "competitor's," answer-choice A would be suggesting that there is an alternative cause for the suffering of the sales, therefore A would not have been eliminated as easily as it would have with the word "competitor's."

The correct answer-choice, answer-choice B, specifies the alternative causes that might've led to the suffering of the sales, "economic factors unrelated to the ad"; whereas the altered version of answer-choice A would be suggesting other unspecified alternative causes for the suffering of the sales, "in the absence of the ad."

Thank you
Mazen
 Adam Tyson
PowerScore Staff
  • PowerScore Staff
  • Posts: 5153
  • Joined: Apr 14, 2011
|
#95466
Does the author take for granted that sales would not have been even lower in the absence of the ad campaign? I don't think so, Mazen. The author might be perfectly willing to concede that the ad did help lead to at least some sales that would not have occurred in the absence of any ad campaign, but still feel that it was ill conceived because sales are down overall compared to last season. Thus, answer A is wrong because it isn't true - the author did not necessarily make that assumption. Since a good Flaw answer must be true (it has to describe something that the author actually did in the course of making their argument), and this answer is not true (the author did not necessarily take this for granted), it cannot be a good answer to a flaw question.

It doesn't matter which ad campaign we are talking about, although the answer is definitely about the one LRG used based on the advice of the competing consultant. The author made no such assumption about their own suggested ad campaign, or any other campaign. The author could believe that any ad campaign, even a terrible one, would lead to at least some sales that would otherwise not have occurred, and still believe that the campaign in question was ill conceived.

The problem in the argument is not that the author fails to consider that the ad campaign may have helped boost sales compared to not having any campaign. It's that they failed to consider other causes for overall sales being down.

I hope that helps clear up any confusion about answer A, but if not and if I have missed the point of your question, please follow up with us again!

Get the most out of your LSAT Prep Plus subscription.

Analyze and track your performance with our Testing and Analytics Package.